Difference between revisions of "1964 AHSME Problems/Problem 30"
Talkinaway (talk | contribs) |
(Added Solution #2) |
||
Line 6: | Line 6: | ||
− | ==Solution== | + | ==Solution 1== |
Dividing the quadratic by <math>7 + 4\sqrt{3}</math> to obtain a monic polynomial will give a linear coefficient of <math>\frac{2 + \sqrt{3}}{7 + 4\sqrt{3}}</math>. Rationalizing the denominator gives: | Dividing the quadratic by <math>7 + 4\sqrt{3}</math> to obtain a monic polynomial will give a linear coefficient of <math>\frac{2 + \sqrt{3}}{7 + 4\sqrt{3}}</math>. Rationalizing the denominator gives: | ||
Line 12: | Line 12: | ||
<math>\frac{(2 + \sqrt{3})(7 - 4\sqrt{3})}{7^2 - 4^2 \cdot 3}</math> | <math>\frac{(2 + \sqrt{3})(7 - 4\sqrt{3})}{7^2 - 4^2 \cdot 3}</math> | ||
+ | <math>=\frac{14 - 12 - \sqrt{3}}{49-48}</math> | ||
− | <math> | + | <math>=2 - \sqrt{3}</math> |
− | |||
− | |||
− | |||
Dividing the constant term by <math>7 + 4\sqrt{3}</math> (and using the same radical conjugate as above) gives: | Dividing the constant term by <math>7 + 4\sqrt{3}</math> (and using the same radical conjugate as above) gives: | ||
Line 22: | Line 20: | ||
<math>\frac{-2}{7 + 4\sqrt{3}}</math> | <math>\frac{-2}{7 + 4\sqrt{3}}</math> | ||
+ | <math>=-2(7 - 4\sqrt{3})</math> | ||
− | <math> | + | <math>=8\sqrt{3} - 14</math> |
− | |||
− | |||
− | |||
So, dividing the original quadratic by the coefficient of <math>x^2</math> gives <math>x^2 + (2 - \sqrt{3})x + 8\sqrt{3} - 14 = 0</math> | So, dividing the original quadratic by the coefficient of <math>x^2</math> gives <math>x^2 + (2 - \sqrt{3})x + 8\sqrt{3} - 14 = 0</math> | ||
Line 34: | Line 30: | ||
<math>\sqrt{(2 - \sqrt{3})^2 - 4(8\sqrt{3} - 14)(1)}</math> | <math>\sqrt{(2 - \sqrt{3})^2 - 4(8\sqrt{3} - 14)(1)}</math> | ||
− | <math>\sqrt{7 - 4\sqrt{3} - 32\sqrt{3} + 56}</math> | + | <math>=\sqrt{7 - 4\sqrt{3} - 32\sqrt{3} + 56}</math> |
+ | |||
+ | <math>=\sqrt{63 - 36\sqrt{3}}</math> | ||
+ | |||
+ | <math>=3\sqrt{7 - 4\sqrt{3}}</math> | ||
+ | |||
+ | Note that if we take <math>\frac{1}{3}</math> of one of the answer choices and square it, we should get <math>7 - 4\sqrt{3}</math>. | ||
+ | The only answers that are (sort of) divisible by <math>3</math> are <math>6 \pm 3\sqrt{3}</math>, so those would make a good first guess. And given that there is a negative sign underneath the radical, <math>6 - 3\sqrt{3}</math> is the most logical place to start. | ||
+ | |||
+ | Since <math>\frac{1}{3}</math> of the answer is <math>2 - \sqrt{3}</math>, and <math>(2 - \sqrt{3})^2 = 7 - 4\sqrt{3}</math>, the answer is indeed <math>\boxed{\textbf{(D)}}</math>. | ||
+ | |||
+ | ==Solution 2== | ||
+ | ===Submitted by MyUsernameWasTaken=== | ||
+ | ====A step-by-step solution==== | ||
+ | |||
+ | By observation, the original equation can be rewritten as | ||
+ | |||
+ | <math>=(4+4\sqrt{3}+3)x^2+(2+\sqrt{3})x-2=0</math> | ||
+ | |||
+ | <math>=(2+\sqrt{3})^2x^2+(2+\sqrt{3})x-2=0</math> | ||
+ | |||
+ | Substituting <math>u = (2+\sqrt{3})x</math>, | ||
+ | |||
+ | <math>u^2+u-2=0</math> | ||
+ | |||
+ | <math>(u-1)(u+2)=0</math> | ||
+ | |||
+ | <math>u=1</math> or <math>u=-2</math> | ||
+ | |||
+ | First root of <math>x</math>: | ||
+ | |||
+ | <math>(2+\sqrt{3})x_1=1</math> | ||
+ | |||
+ | <math>x_1=\frac{1}{2+\sqrt{3}}</math> | ||
+ | |||
+ | <math>x_1=2-\sqrt{3}</math> | ||
+ | |||
+ | Second root of <math>x</math>: | ||
+ | |||
+ | <math>(2+\sqrt{3})x_2=-2</math> | ||
+ | |||
+ | <math>x_2=\frac{-2}{2+\sqrt{3}}</math> | ||
+ | |||
+ | <math>x_2=-2(2-\sqrt{3})</math> | ||
+ | |||
+ | <math>x_2=-4+2\sqrt{3}</math> | ||
+ | |||
+ | Now, to find which root of <math>x</math> is larger: | ||
+ | |||
+ | Assume that <math>2-\sqrt{3}>-4+2\sqrt{3}</math>. | ||
+ | |||
+ | <math>6>3\sqrt{3}</math> | ||
+ | |||
+ | <math>2>\sqrt{3}</math> | ||
+ | |||
+ | <math>\sqrt{4}>\sqrt{3}</math> which is true. Hence, the first root of <math>x</math> is the larger one. | ||
+ | |||
+ | Finally, finding the difference between the larger and smaller roots of <math>x</math>: | ||
− | <math> | + | <math>x_1-x_2</math> |
− | <math> | + | <math>=(2-\sqrt{3})-(-4+2\sqrt{3})</math> |
− | + | <math>=6-3\sqrt{3}</math> | |
− | + | Therefore, the answer is <math>\boxed{\textbf{(D)}}</math>. | |
==See Also== | ==See Also== |
Revision as of 13:39, 24 March 2023
Contents
Problem
The larger root minus the smaller root of the equation is
Solution 1
Dividing the quadratic by to obtain a monic polynomial will give a linear coefficient of . Rationalizing the denominator gives:
Dividing the constant term by (and using the same radical conjugate as above) gives:
So, dividing the original quadratic by the coefficient of gives
From the quadratic formula, the positive difference of the roots is . Plugging in gives:
Note that if we take of one of the answer choices and square it, we should get . The only answers that are (sort of) divisible by are , so those would make a good first guess. And given that there is a negative sign underneath the radical, is the most logical place to start.
Since of the answer is , and , the answer is indeed .
Solution 2
Submitted by MyUsernameWasTaken
A step-by-step solution
By observation, the original equation can be rewritten as
Substituting ,
or
First root of :
Second root of :
Now, to find which root of is larger:
Assume that .
which is true. Hence, the first root of is the larger one.
Finally, finding the difference between the larger and smaller roots of :
Therefore, the answer is .
See Also
1964 AHSC (Problems • Answer Key • Resources) | ||
Preceded by Problem 29 |
Followed by Problem 31 | |
1 • 2 • 3 • 4 • 5 • 6 • 7 • 8 • 9 • 10 • 11 • 12 • 13 • 14 • 15 • 16 • 17 • 18 • 19 • 20 • 21 • 22 • 23 • 24 • 25 • 26 • 27 • 28 • 29 • 30 • 31 • 32 • 33 • 34 • 35 • 36 • 37 • 38 • 39 • 40 | ||
All AHSME Problems and Solutions |
The problems on this page are copyrighted by the Mathematical Association of America's American Mathematics Competitions.